PRACTICE TEST 3 - PRACTICE TESTS WITH DETAILED ANSWER KEYS - SAT Test Prep

SAT Test Prep

CHAPTER 16
PRACTICE TESTS WITH DETAILED ANSWER KEYS

PRACTICE TEST 3

ANSWER SHEET

Directions for Test

• Remove these answer sheets from the book and use them to record your answers to this test.

• This test will require 3 hours and 20 minutes to complete. Take this test in one sitting.

• The time allotment for each section is written clearly at the beginning of each section. This test contains six 25-minute sections, two 20-minute sections, and one 10-minute section.

• This test is 25 minutes shorter than the actual SAT, which will include a 25-minute “experimental” section that does not count toward your score. That section has been omitted from this test.

• You may take one short break during the test, of no more than 10 minutes in length.

• You may only work on one section at any given time.

• You must stop ALL work on a section when time is called.

• If you finish a section before the time has elapsed, check your work on that section. You may NOT work on any other section.

• Do not waste time on questions that seem too difficult for you.

• Use the test book for scratchwork, but you will receive credit only for answers that are marked on the answer sheets.

• You will receive one point for every correct answer.

• You will receive no points for an omitted question.

• For each wrong answer on any multiple-choice question, your score will be reduced by ¼ point.

• For each wrong answer on any “numerical grid-in” question, you will receive no deduction.

When you take the real SAT, you will be asked to fill in your personal information in grids as shown below.

Start with number 1 for each new section. If a section has fewer questions than answer spaces, leave the extra answer spaces blank. Be sure to erase any errors or stray marks completely.

CAUTION

Use the answer spaces in the grids below for Section 2 or Section 3 only if you are told to do so in your test book.

Student-Produced Responses

Start with number 1 for each new section. If a section has fewer questions than answer spaces, leave the extra answer spaces blank. Be sure to erase any errors or stray marks completely.

CAUTION

Use the answer spaces in the grids below for Section 4 or Section 5 only if you are told to do so in your test book.

Student-Produced Responses

Start with number 1 for each new section. If a section has fewer questions than answer spaces, leave the extra answer spaces blank. Be sure to erase any errors or stray marks completely.

CAUTION

Use the answer spaces in the grids below for Section 6 or Section 7 only if you are told to do so in your test book.

Student-Produced Responses

Start with number 1 for each new section. If a section has fewer questions than answer spaces, leave the extra answer spaces blank. Be sure to erase any errors or stray marks completely.

ESSAY
Time—25 minutes


Write your essay on separate sheets of standard lined paper.


The essay gives you an opportunity to show how effectively you can develop and express ideas. You should therefore take care to develop your point of view, present your ideas logically and clearly, and use language precisely.

Your essay must be written on the lines provided on your answer sheet—you will receive no other paper on which to write. You will have enough space if you write on every line, avoid wide margins, and keep your handwriting to a reasonable size. Remember that people who are not familiar with your handwriting will read what you write. Try to write or print so that what you are writing is legible to those readers.

Important reminders:

A pencil is required for the essay. An essay written in ink will receive a score of zero.

Do not write your essay in your test book. You will receive credit only for what you write on your answer sheet.

An off-topic essay will receive a score of zero.

You have 25 minutes to write an essay on the topic assigned below.


Consider carefully the issue discussed in the following passage, then write an essay that answers the question posed in the assignment.


Many among us like to blame violence and immorality in the media for a “decline in morals” in society. Yet these people seem to have lost touch with logic. Any objective examination shows that our society is far less violent or exploitative than virtually any society in the past. Early humans murdered and enslaved each other with astonishing regularity, without the help of gangsta rap or Jerry Bruckheimer films.


Assignment: Do violence and immorality in the media make our society more dangerous and immoral? Write an essay in which you answer this question and discuss your point of view on this issue. Support your position logically with examples from literature, the arts, history, politics, science and technology, current events, or your experience or observation.


If you finish before time is called, you may check your work on this section only. Do not turn to any other section in the test.

SECTION 2
Time—25 minutes 20 questions


Turn to Section 2 of your answer sheet to answer the questions in this section.



Directions: For this section, solve each problem and decide which is the best of the choices given. Fill in the corresponding circle on the answer sheet. You may use any available space for scratchwork.



Notes

1. The use of a calculator is permitted.

2. All numbers used are real numbers.

3. Figures that accompany problems in this test are intended to provide information useful in solving the problems. They are drawn as accurately as possible EXCEPT when it is stated in a specific problem that the figure is not drawn to scale. All figures lie in a plane unless otherwise indicated.

4. Unless otherwise specified, the domain of any function f is assumed to be the set of all real numbers x for which f (x) is a real number.



Reference Information

The number of degrees of arc in a circle is 360.

The sum of the measures in degrees of the angles of a triangle is 180.


1. If , what is the value of x?

(A) 3

(B) 7

(C) 11

(D) 17

(E) 25

2. Erica spends $.95 each day for her newspaper subscriptions. She would like to determine the approximate amount she spends during the month of July, which has 31 days. Which of the following would provide her with the best estimate?

(A) $.50 × 30

(B) $1.00 × 30

(C) $1.50 × 30

(D) $.50 × 35

(E) $1.00 × 35

3. In the figure above, lines l, m, and n intersect in a single point. What is the value of w + x?

(A) 40

(B) 70

(C) 90

(D) 130

(E) 140

4. Let the function g be defined by the equation . What is the value of g (5)?

(A) 8

(B) 11

(C) 15

(D) 19

(E) 23

5. If , which of the following equations expresses the fact that when the difference between x and y is multiplied by their sum, the product is 18?

6. If , what is the value of x?

(A) 3

(B) 9

(C) 27

(D) 36

(E) 81

7. Chris buys a chocolate bar and a pack of gum for $1.75. If the chocolate bar costs $.25 more than the pack of gum, how much does the pack of gum cost?

(A) $.25

(B) $.50

(C) $.75

(D) $1.00

(E) $1.50

8. 40% of 80 is what percent of 96?

(A) 20%

(B) 30%

(C) 33%

(D) 50%

(E) 66%

9. If l, m, and n are positive integers greater than 1, , and , then which of the following must be true?

10. According to the graph above, ABC Company showed the greatest change in profits between which 2 years?

(A) 1996 and 1997

(B) 1997 and 1998

(C) 1998 and 1999

(D) 1999 and 2000

(E) 2000 and 2001

11. In a 9th-grade class, 12 students play soccer, 7 students play tennis, and 9 students play lacrosse. If 4 students play exactly two of the three sports and all other students play only one, how many students are in the class?

(A) 28

(B) 24

(C) 20

(D) 18

(E) 16

12. The point (14, 14) is the center of a circle, and (2, 9) is a point on the circle. What is the length of the diameter of the circle?

(A) 24

(B) 26

(C) 50

(D) 144π

(E) 169π

13. The population of Boomtown doubles every 18 months. In January of 2000, its population was exactly 12,000. At this rate, approximately when should the population reach 96,000?

(A) January 2003

(B) July 2004

(C) January 2006

(D) July 2007

(E) January 2012

14. In how many different ways can five students of different heights be arranged in a line if the tallest student cannot be on either end?

(A) 24

(B) 25

(C) 72

(D) 96

(E) 120

15. In the figure above, and . If a, b, and c are all integers, what is the greatest possible value of b?

(A) 43

(B) 46

(C) 60

(D) 86

(E) 89

16. In the figure above, ΔACF is equilateral, with sides of length 4. If B, D, and E are the midpoints of their respective sides, what is the sum of the areas of the shaded regions?

17. Given the graph of above, which of the following sets represents all values of x for which ?

(A) all real numbers

18. If a is a number chosen randomly from set X and b is a number chosen randomly from set Y, what is the probability that ab is greater than 20 but less than 50?

19. If and , what is the value of a + b?

(A) 6

(B) 7

(C) 11

(D) 12

(E) 13

20. Given the graph of y = f (x) above, which of the following represents the graph of ?

If you finish before time is called, you may check your work on this section only. Do not turn to any other section of the test.

SECTION 3
Time—25 minutes
24 questions


Turn to Section 3 of your answer sheet to answer the questions in this section.



Directions: For each question in this section, select the best answer from among the choices given and fill in the corresponding circle on the answer sheet.



Each sentence below has one or two blanks, each blank indicating that something has been omitted. Beneath the sentence are five words or sets of words labeled A through E. Choose the word or set of words that, when inserted in the sentence, best fits the meaning of the sentence as a whole.

EXAMPLE:

Rather than accepting the theory unquestioningly, Deborah regarded it with-----.

(A) mirth

(B) sadness

(C) responsibility

(D) ignorance

(E) skepticism


1. Although he purchased his computer only 10 months ago, rapid improvements in technology have left Raúl with ------- machine.

(A) an obsolete

(B) an adjunct

(C) a novel

(D) an automated

(E) an elusive

2. Only if the number of applicants continues to -------- can the admissions committee justify offering more scholarships in order to increase the number of applications.

(A) mushroom

(B) expand

(C) plummet

(D) satiate

(E) burgeon

3. My father is so ------- that he will never even consider another person”s viewpoint to be valid if it is different from his own.

(A) pragmatic

(B) dogmatic

(C) phlegmatic

(D) cordial

(E) curt

4. J. K. Rowling”s Harry Potter series is a collection of works that are ------- for children but are still ------- to adults.

(A) penned . . prosaic

(B) employed . . morose

(C) censored . . incongruous

(D) designed . . tedious

(E) authored . . engaging

5. Julia approaches her homework assignments in such ------- way that it is very difficult to believe that she is at the top of her class.

(A) an adept

(B) a diligent

(C) a fanatical

(D) an extroverted

(E) a laggardly

6. The President was such a ------- orator that his opponents were always supremely cautious about agreeing to debate him.

(A) redoubtable

(B) staid

(C) magnanimous

(D) weak

(E) stoic

7. The newest clothing line revealed at the show was an eclectic mix that ranged from the modest and unadorned to the ------- and garish.

(A) austere

(B) prophetic

(C) cordial

(D) ostentatious

(E) solitary

8. Neil Campbell”s textbook Biology is ------- and yet -------; it includes all of the essential information without ever being verbose.

(A) compendious . . circumlocutory

(B) reprehensible . . terse

(C) comprehensive . . concise

(D) praiseworthy . . grandiloquent

(E) painstaking . . redundant


The passages below are followed by questions based on their content; questions following a pair of related passages may also be based on the relationship between the paired passages. Answer the questions on the basis of what is stated or implied in the passage and in any introductory material that may be provided.


Questions 9–12 are based on the following passages.

9. The word “unrivaled” in line 5 most nearly means

(A) without enemies

(B) supremely abundant

(C) militarily superior

(D) unimaginable

(E) highly intelligent

10. Which of the following best describes the contrast between the “people” (line 9) as characterized in Passage 1 and the “citizens” (line 20) as characterized in Passage 2?

(A) the “people” are ignorant, while the “citizens” are well educated

(B) the “people” lack fortitude, while the “citizens” are courageous

(C) the “people” are worldly, while the “citizens” are parochial

(D) the “people” are proud of their leaders, while the “citizens” are not

(E) the “people” lack unity, while the “citizens” lack awareness

11. Passage 1 makes all of the following claims about the state of society EXCEPT that

(A) an increasing number of people are happy with their lives

(B) information is disseminated more rapidly than in the past

(C) the current economy is strong

(D) social inequities are deepening

(E) workers” incomes are not increasing

12. Unlike the author of Passage 1, the author of Passage 2 does which of the following?

(A) contrasts an ideal with a reality

(B) explains a study

(C) compares the past with the present

(D) describes an injustice

(E) acknowledges a responsibility

Questions 13–19 are based on the following passage.

The following passage is adapted from a short story published by a Russian author in the late 1970s.

13. Which of the following best describes the characterization of the man and the woman in the first two paragraphs?

(A) He is confused, while she is passionate.

(B) He is angry, while she is jocular.

(C) He is stoic, while she is serene.

(D) He is ambivalent, while she is anxious.

(E) He is disdainful, while she is whimsical.

14. The author suggests that one “who is suddenly overwhelmed by terror” (lines 9–10) is temporarily

(A) vindictive

(B) defensive

(C) cautious

(D) disoriented

(E) resentful

15. The description of “the catastrophe” (lines 11–12) serves primarily to suggest that

(A) the couple has endured a terrible accident

(B) Ognev is devastated by Vera”s harsh words

(C) Ognev is deeply troubled by Vera”s passionate expression of love

(D) Ognev holds Vera responsible for a crime

(E) Vera has told Ognev a horrible secret

16. In line 24, “objects” most nearly means

(A) possessions

(B) facts

(C) decorations

(D) goals

(E) complaints

17. The passage suggests that the “bad and strange” (line 39) thing that was taking place in Ognev”s heart was his

(A) eagerness

(B) sadism

(C) jealousy

(D) hatred

(E) disaffection

18. In lines 57–58, “statistics, books or philosophical truths” are mentioned as examples of things that

(A) Vera does not understand

(B) Ognev and Vera share reluctantly

(C) Ognev abandoned long ago

(D) Vera loves passionately

(E) Ognev inexplicably values more highly than passion

19. The primary function of the final paragraph is to show Ognev”s

(A) struggle to understand his own feelings

(B) anger about Vera”s misrepresentation of her feelings

(C) frustration with the voices in his head

(D) outrage with his inability to understand a philosophical concept

(E) appreciation of Vera”s beauty

Questions 20–24 are based on the following passage.

The following is part of an introduction to the publication of a speech delivered by President Lyndon B. Johnson in the 1960s.

20. In the first paragraph, the marchers are characterized as

(A) ruthless

(B) gleeful

(C) intellectual

(D) stoic

(E) shocked

21. The passage indicates that the 1964 Civil Rights Act was deficient in that it did not

(A) sufficiently pressure local officials to extend voting privileges to all citizens

(B) provide enough funds to promote voter registration drives

(C) punish felons who committed hate crimes

(D) provide military protection for the Selma marchers

(E) invest in minority-owned businesses

22. In line 55, shattering most nearly means

(A) exploding

(B) disturbing

(C) fragmenting

(D) violent

(E) loud

23. The quotation in lines 57–59 (“They never seemed … in their eyes”) indicates that Johnson

(A) understood the political process at a young age

(B) was unfamiliar with Mexican-American customs

(C) empathized strongly with his students

(D) was a victim of bigotry

(E) was unaware of the difficulties his students faced

24. The passage indicates that Johnson, unlike previous presidents, handled the issue of civil rights by

(A) successfully integrating the issue into his reelection campaign

(B) approaching the cause with objectivity and impartiality

(C) speaking clearly to reporters using terms they wanted to hear

(D) focusing primarily on the Mexican-American population

(E) directly addressing the public on the issue and describing it in personal terms

If you finish before time is called, you may check your work on this section only. Do not turn to any other section of the test.

SECTION 4
Time—25 minutes
35 questions


Turn to Section 4 of your answer sheet to answer the questions in this section.



Directions: For each question in this section, select the best answer from among the choices given and fill in the corresponding circle on the answer sheet.



The following sentences test correctness and effectiveness of expression. Part of each sentence or the entire sentence is underlined; beneath each sentence are five ways of phrasing the underlined material. Choice A repeats the original phrasing; the other four choices are different. If you think the original phrasing produces a better sentence than any of the alternatives, select choice A; if not, select one of the other choices.

In making your selection, follow the requirements of standard written English; that is, pay attention to grammar, choice of words, sentence construction, and punctuation. Your selection should result in the most effective sentence—clear and precise, without awkwardness or ambiguity.

EXAMPLE:

The children couldn”t hardly believe their eyes.

(A) couldn”t hardly believe their eyes

(B) could hardly believe their eyes

(C) would not hardly believe their eyes

(D) couldn”t nearly believe their eyes

(E) couldn”t hardly believe his or her eyes


1. Exhausted from a day of hiking across steep, rain-soaked paths, the group of campers were relieved upon the final reaching of the car.

(A) group of campers were relieved upon the final reaching of the car

(B) camping group became relieved after they got to the car

(C) group of campers was relieved to finally reach the car

(D) campers were relieved after the car was finally reached

(E) group was relieved after the campers finally reached the car

2. Theodore Roosevelt”s first term as President was marked by a ferocious battle between labor and management in Pennsylvania”s coal mining industry.

(A) was marked by a ferocious battle between labor and management

(B) was marked by a ferocious battle of labor”s and management”s

(C) saw a ferocious battle: between labor and management

(D) was marked ferociously by labor and management”s battle

(E) was marking a ferocious battle between labor and management

3. Many great scientists and inventors of the past, such as Nikola Tesla, has possessed the ability of extraordinary visualization skills that enabled them to analyze the most minute details of complex machines before the devices were even constructed.

(A) has possessed the ability of extraordinary visualization skills

(B) have been able to possess extraordinary visualization skills

(C) possessed skills in visualization that was extraordinary

(D) possessed extraordinary visualization skills

(E) possessed skills of visualizing that was extraordinary

4. The Thracians, originally divided into numerous tribes, came together politically under King Teres in 500 BC, and it enabled their resistance against the many Roman invasions that would follow in the centuries to come.

(A) it enabled their resistance against

(B) this unity enabled them to resist

(C) enabling the ability to resist

(D) that enabled them to resist

(E) this unity gave them the ability of resisting

5. Disillusioned by American politics and culture, Ernest Hemingway led an exodus of expatriate authors on an overseas journey across the Atlantic following the World War I.

(A) led an exodus of expatriate authors on an overseas journey

(B) took an overseas journey leading an exodus of expatriate authors

(C) led an exodus of expatriate authors

(D) has led an exodus of expatriate authors

(E) leading an exodus of expatriate authors

6. Walter Cronkite was known for his honest presentation of the news, plus the ability to be reassuring with his tone.

(A) news, plus the ability to be reassuring with his tone

(B) news, plus his reassuring tone

(C) news plus the reassuring nature of his tone

(D) news and his tone that was reassuring

(E) news and his reassuring tone

7. Only half as many students study computer science than they did just a decade ago.

(A) than they did

(B) than was true

(C) as did

(D) when compared to

(E) than

8. Auto racing, often thought of as a regional phenomenon, therefore is quite popular throughout the nation.

(A) therefore is quite popular

(B) henceforth is quite popular

(C) is thus quite popular

(D) is actually quite popular

(E) in retrospect, is quite popular

9. The band decided to allow downloading their songs for their fans free of charge, in the hope of increasing its popularity.

(A) downloading their songs for their fans free of charge

(B) their fans downloading their songs free of charge

(C) its fans to download its songs free of charge

(D) free downloading of their songs to its fans

(E) downloading of its songs to its fans, which were free of charge

10. The most likely reasons for the recent surge in legislation is the fact that the voters agree on the issues and the political parties stopping bickering.

(A) is the fact that the voters agree on the issues and the political parties stopping

(B) are because the voters agree on the issues and the political parties have stopped

(C) are that the voters agree on the issues and that the political parties have stopped

(D) is the voters agreeing on the issues and the political parties stopping

(E) are the voters agreeing on the issues and the political parties have stopped

11. An untiring defender of the downtrodden, Clarence Darrow”s oratory could mesmerize his audiences and devastate his opponents.

(A) Clarence Darrow”s oratory could mesmerize his audiences and devastate his opponents

(B) Clarence Darrow could mesmerize his audiences and devastate his opponents with his oratory

(C) the oratory of Clarence Darrow could mesmerize his audiences and devastate his opponents

(D) Clarence Darrow”s audiences could be mesmerized by his oratory and his opponents devastated by it

(E) Clarence Darrow could mesmerize his audiences with his oratory, and his opponents could be devastated by it


The following sentences test your ability to recognize grammar and usage errors. Each sentence contains either a single error or no error at all. No sentence contains more than one error. The error, if there is one, is underlined and lettered. If the sentence contains an error, select the one underlined part that must be changed to make the sentence correct. If the sentence is correct, select choice E. In choosing answers, follow the requirements of standard written English.

EXAMPLE:

By the time the halfway point begun to hit their stride.


12. The local dairy company is in the state, it is surprising that the delivery of our milk products over the last few days late.

13. Sea turtle hatchlings can find their way by sight alone, even at night, they are capable visually between the bright reflections from the ocean surface the dark silhouettes of sand dunes and vegetation.

14. several members of the committee sponsoring a dinner to raise money for their to encourage driving.

15. The lavish photographs and fascinating diagrams so engaging that I seriously becoming

16. Behavioral scientists that the way chimpanzees and alliances is

17. When the window the of the cool spring breeze were by the

18. The probability by lightning the probability the lottery, minuscule.

19. the new editorial guidelines for publication, before an author a manuscript to the publisher, must first by a qualified content expert.

20. Night, Elie Wiesel employs a disjointed style, shifting point of view in order the fragmented nature of ghetto life in Germany during the World War II.

21. we had expected poor service at the resort, more than the attention throughout our stay.

22. After we a leisurely meal, we walked and a jazz club where a talented young trio

23. Ancient Babylonian physicians among the first the character and course of diseases scientifically, they frequently attributed the causes of those ailments of gods or demons.

24. When of the angler fish its jaws, with bands of sharp inward-pointing teeth, triggered to snap shut.

25. Some doctors taking vitamins decrease a patient”s susceptibility

26. When my parents out to dinner, they left me the our babysitter, next door to us.

27. Since 2001, the company employee training than in the previous 10 years

28. a productive working relationship high school students, should not only command respect, but develop a productive rapport.

29. After several trials, the chemists that the precipitates more effectively separated a high-speed centrifuge by a filtration system.


Directions: The following passage is an early draft of an essay. Some parts of the passage need to be rewritten.

Read the passage and select the best answers for the questions that follow. Some questions are about particular sentences or parts of sentences and ask you to improve sentence structure or word choice. Other questions ask you to consider organization and development. In choosing answers, follow the requirements of standard written English.


Questions 30–35 refer to the following passage.

(1) While known when he was the President for his abundant energy and muscular build as an adult, Theodore Roosevelt”s build as a child was actually quite puny. (2) Stricken with asthma, he was taught early that strenuous physical activity might be dangerous to his health and that, in fact, it might even be fatal. (3) Determined to overcome this obstacle, Roosevelt trained his body relentlessly and built his impressive girth through sheer grit and determination. (4) That these childhood passions stayed with him throughout his adult life should not be surprising. (5) Physical activities, though, were not the only childhood fascination to play a prominent role later in his life.

(6) A skilled hunter, Roosevelt spent much of his leisure time hunting various forms of game. (7) Beginning during his undergraduate days at Harvard, he spent significant time in snow-covered Maine forests as well as the arid deserts of the Dakota territory. (8) As a child, Theodore was so enraptured by birds, he would spend hours observing and writing about them, even phonetically spelling out their various calls and songs. (9) Upon reaching government office, Roosevelt became the first true conservationist, pushing for laws to protect wildlife and resources. (10) He cherished nature in all its forms, seeking to understand its variety through research and experience.

(11) By openly maintaining these passions while in political office, Roosevelt redefined the role of the American politician. (12) While his predecessors had often been aloof with regard to their own personal feelings, Roosevelt advertised his sense of morality by talking openly about it repeatedly with citizens and reporters in speeches and newspapers. (13) In the dawning of a new, industrialized age, Roosevelt chose to take on controversial issues, battling through the spoils system, disputes between management and labor, and the question of imperialism.

30. In context, which of the following is the best revision of sentence 1 (reproduced below)?

While known when he was the President for his abundant energy and muscular build as an adult, Theodore Roosevelt”s build as a child was actually quite puny.

(A) While Theodore Roosevelt was known for his energy and muscular build, but the President was actually a quite puny child.

(B) Although known for his abundant energy and muscular build as an adult, President Theodore Roosevelt was actually quite puny as a child.

(C) While puny as a child, Theodore Roosevelt was known for his abundant energy and muscular build while being President.

(D) As President, Theodore Roosevelt was known for his abundant energy and muscular build, not for being puny as a child.

(E) Theodore Roosevelt was puny as a child and was known for his abundant energy and muscular build as President.

31. In context, which of the following is the best revision of the underlined portion of sentence 3 (reproduced below)?

Determined to overcome this obstacle, Roosevelt trained his body relentlessly and built his impressive girth through sheer grit and determination.

(A) (no revision needed)

(B) that this obstacle should be overcome

(C) to overcome such ideas that became obstacles

(D) not to allow this to become an obstacle standing in his way

(E) to take obstacles out of his way

32. Where is the most appropriate place to move sentence 4?

(A) Before sentence 1

(B) Before sentence 2

(C) Before sentence 6, to start the second paragraph

(D) After sentence 10, to end the second paragraph

(E) After sentence 13

33. Which of the following provides the most logical ordering of the sentences in paragraph 2?

(A) 7, 9, 10, 6, 8

(B) 8, 10, 7, 6, 9

(C) 8, 10, 9, 6, 7

(D) 9, 7, 8, 10, 6

(E) 7, 10, 8, 6, 9

34. If the author wanted to make sentence 7 more specific, which of the following details would fit best in the context of the second paragraph?

(A) Roosevelt”s age

(B) information about Roosevelt”s course of study

(C) details of Roosevelt”s activities in the deserts and forests

(D) an explanation of why the climate of Maine is so different from the climate of the Dakota territory

(E) information about Roosevelt”s political affiliation prior to these excursions

35. Where is the best place to insert the following sentence?

His brazen moves were often criticized, but Theodore Roosevelt will go down in the annals of history as a man who was always true to himself, whether as a private citizen or as President of the United States.

(A) Before sentence 1

(B) After sentence 1

(C) After sentence 5

(D) Before sentence 11

(E) After sentence 13

If you finish before time is called, you may check your work on this section only. Do not turn to any other section of the test.

SECTION 5
Time—25 minutes
18 questions


Turn to Section 5 of your answer sheet to answer the questions in this section.



Directions: This section contains two types of questions. You have 25 minutes to complete both types. For questions 1–8, solve each problem and decide which is the best of the choices given. Fill in the corresponding circle on the answer sheet. You may use any available space for scratchwork.



Notes

1. The use of a calculator is permitted.

2. All numbers used are real numbers.

3. Figures that accompany problems in this test are intended to provide information useful in solving the problems. They are drawn as accurately as possible EXCEPT when it is stated in a specific problem that the figure is not drawn to scale. All figures lie in a plane unless otherwise indicated.

4. Unless otherwise specified, the domain of any function f is assumed to be the set of all real numbers x for which f (x) is a real number.



Reference Information

The number of degrees of arc in a circle is 360.

The sum of the measures in degrees of the angles of a triangle is 180.


1. In the figure above, what is the value of 2x?

(A) 36

(B) 72

(C) 90

(D) 108

(E) 132

2. If , then x could be

(A) –6

(B) –2

(C) 0

(D) 4

(E) 6

3. In the figure above, what is the value of ?

(A) 56

(B) 128

(C) 256

(D) 264

(E) 322

4. If f (x) =x2 – 4, for what positive value of x does f (x) = 32?

(A) 5

(B) 6

(C) 7

(D) 8

(E) 9

5. A can of mixed nuts contains cashews, almonds, peanuts, and walnuts in the ratio of 2 to 4 to 5 to 7, respectively, by weight. What fraction of the mixture by weight is almonds?

6.Twenty students in a chemistry class took a test on which the overall average score was 75. If the average score for 12 of those students was 83, what was the average score for the remaining members of the class?

(A) 60

(B) 61

(C) 62

(D) 63

(E) 64

7. In the figure above, the vertices of square EFGH are on the diagonals of square ABCD. If and what is the sum of the lengths (heavier lines)?

(A) 24

(B) 28

(C) 32

(D) 36

(E) 38

8. In the correctly worked addition problem above, each letter represents a different nonzero digit. What is the value of 2R + T?

(A) 4

(B) 5

(C) 10

(D) 11

(E) 13


Directions: For student-produced response questions 9–18, use the grids at the bottom of the answer sheet page on which you have answered questions 1–8.


Each of the remaining ten questions requires you to solve the problem and enter your answer by marking the circles in the special grid, as shown in the examples below. You may use any available space for scratchwork.

• Mark no more than one circle in any column.

• Because the answer sheet will be machine-scored, you will receive credit only if the circles are filled in correctly.

• Although not required, it is suggested that you write your answer in the boxes at the top of the columns to help you fill in the circles accurately.

• Some problems may have more than one correct answer. In such cases, grid only one answer.

• No question has a negative answer.

Mixed numbers such as 3; must be gridded as 3.5 or 7/2. (If is gridded, it will be interpreted as not )

Decimal Answers: If you obtain a decimal answer with more digits than the grid can accommodate, it may be either rounded or truncated, but it must fill the entire grid. For example, if you obtain an answer such as 0.6666…, you should record your result as .666 or .667. A less accurate value such as .66 or .67 will be scored as incorrect.

Acceptable ways to grid are:

9. For all real numbers n, let be defined by What is the value of?

10. The Civics Club earned 25% more at its bake sale in 2007 than it did in 2006. If it earned $600 at its bake sale in 2006, how much did it earn at its bake sale in 2007?

11. If the sum of two numbers is 4 and their difference is 2, what is their product?

12. In rectangle LMNO above, P is the midpoint of side . If the perimeter of the rectangle is 48 and side is twice the length of side , what is the area of the shaded region?

13. If , what is the value of x?

14. Points P, Q, R, and S lie on a line in that order. If is twice as long as and four times as long , what is the value of

15. The figure above shows the graph in the xy-plane of a quadratic function with a vertex at (m, n). What is the value of m?

16. If the sum of five consecutive even integers is 110, what is the least of these integers?

17. According to the data in the table above, by what percent did the number of applicants to Collins College increase from 1990 to 1995? (Disregard the % symbol when entering your answer into the grid. For instance, grid 50% as 50.)

18. A jar contains only black, white, and red marbles. If randomly choosing a black marble is four times as likely as randomly choosing a white marble, and randomly choosing a red marble is five times as likely as randomly choosing a black marble, then what is the smallest possible number of marbles in the jar?

If you finish before time is called, you may check your work on this section only. Do not turn to any other section of the test.

SECTION 6
Time—25 minutes
24 questions


Turn to Section 6 of your answer sheet to answer the questions in this section.



Directions: For each question in this section, select the best answer from among the choices given and fill in the corresponding circle on the answer sheet.



Each sentence below has one or two blanks, each blank indicating that something has been omitted. Beneath the sentence are five words or sets of words labeled A through E. Choose the word or set of words that, when inserted in the sentence, best fits the meaning of the sentence as a whole.

EXAMPLE:

Rather than accepting the theory unquestioningly, Deborah regarded it with -----.

(A) mirth

(B) sadness

(C) responsibility

(D) ignorance

(E) skepticism


1. If John had not been there to ------- when tensions began to rise at the meeting, a fight would surely have ensued.

(A) intervene

(B) coalesce

(C) harass

(D) intermingle

(E) exacerbate

2. The defendant hoped that the testimony of the surprise witness would corroborate his alibi and ------- him of the crime of which he had been accused.

(A) convoke

(B) synthesize

(C) impeach

(D) absolve

(E) magnify

3. Rachel”s ------- driving is not surprising, given that she spends ------- hours each day ensnarled in traffic delays.

(A) antipathy for . . delightful

(B) penchant for . . uncountable

(C) predilection for . . dreary

(D) proclivity for . . desperate

(E) aversion to . . insufferable

4. Many medical practices once considered “state of the art” are now thought to be ------- by physicians who are often incredulous that such barbaric acts were once -------.

(A) primitive . . sanctioned

(B) ingenious . . approved

(C) boorish . . censured

(D) innovative . . endorsed

(E) foolhardy . . condemned

5. The Prime Minister had vetoed the proposal several times in the past; thus, it came as a great surprise to the public when he ------- the same law in his most recent speech.

(A) articulated

(B) sanctioned

(C) denounced

(D) initiated

(E) abbreviated


The passages below are followed by questions based on the content. Answer the questions on the basis of what is stated or implied in the passage and in any introductory material that may be provided.


Questions 6 and 7 are based on the following passage.

6. Based on the information in the passage, with which of the following statements would the author most likely agree?

(A) Navajo warriors obey their wives obsequiously.

(B) Birds are a particularly vital food source for the Navajo.

(C) A Navajo man who disrespects a woman would likely face censure.

(D) The Navajo do not believe in reincarnation.

(E) In the winter, the Navajo migrate to warmer climates.

7. The word “administer” in line 6 most nearly means

(A) manage

(B) maintain

(C) govern

(D) rehearse

(E) dispense

Questions 8 and 9 are based on the following passage.

8. As used in line 6, “palliative” most nearly means

(A) punitive

(B) remedial

(C) analgesic

(D) curative

(E) altruistic

9. The passage suggests that in cases of extreme illness, doctors may have difficulty in determining their patients”

(A) state of mind

(B) prognosis

(C) quality of life

(D) tolerance of pain

(E) ability to remember facts

First passage: “The Navajo Indians,” William M. Edwardy, Harper”s Weekly, July 1890.

Second passage: Copyright 2004 Mark Anestis. All rights reserved.

Questions 10–16 are based on the following passage.

The following passage is excerpted from a recent book about seismology, the study of earthquakes.

10. Which of the following is the best title for this passage?

(A) The Effects of Earthquakes on Ground-Water

(B) The Search for Earthquake Precursors

(C) A Novel Theory of the Origin of Earthquakes

(D) A History of Chinese Earthquakes

(E) How Animals Anticipate Earthquakes

11. The passage indicates that foreshocks are “not … very useful” (lines 9–10) in predicting earthquakes because they

(A) are exceptionally difficult to detect

(B) occur simultaneously with changes in ground-water

(C) are not part of the theory of dilatancy

(D) interfere with electrostatic phenomena

(E) are impossible to distinguish from earthquakes themselves

Excerpted from Furious Earth, by Ellen J. Prager, McGraw-Hill, New York, 2000. Reproduced with permission of The McGraw-Hill Companies.

12. According to the passage, which of the following features of ground-water have been reported to change immediately prior to an earthquake (lines 16–20)?

I. density

II. clarity

III. flow

(A) II only

(B) III only

(C) I and II only

(D) I and III only

(E) I, II, and III

13. Which of the following could be considered a logical inconsistency in the passage?

(A) The passage states that foreshocks are not useful predictors of earthquakes but then cites foreshocks as instrumental to predicting an earthquake.

(B) The passage says that the Chinese are interested in predicting earthquakes but then says that they were devastated by the Tangshan earthquake.

(C) The passage reports that animals behaved strangely before an earthquake but then attributes this behavior to electrostatic phenomena.

(D) The passage states that the town of Haicheng was safely evacuated but then says that its citizens were forced to watch outdoor movies.

(E) The passage suggests that both strain and creep could be considered earthquake precursors.

14. Which of the following best describes the function of the third paragraph?

(A) to describe an application of a theory

(B) to provide an alternative perspective

(C) to recount a scientific experiment

(D) to summarize the ancient origins of a theory

(E) to demonstrate the difficulties of employing a technique

15. The passage suggests that the Tangshan earthquake

(A) was caused by strain and creep

(B) was preceded by changes in the ground-water

(C) caused more damage than the Haicheng earthquake did

(D) was preceded by several foreshocks

(E) was anticipated by the theory of dilatancy

16. In line 46, the word “evacuation” is placed in quotations in order to

(A) imply that an action was ineffective

(B) indicate that it is an archaic term

(C) emphasize the primitiveness of Chinese scientific methods

(D) suggest that a certain practice was unconventional

(E) underscore that an action was intended, but not implemented

Questions 17–24 are based on the following passage.

The following passage contains an excerpt taken from an anthology of autobiographies of American women.

17. Immediately upon arriving in America, the author was cared for by

(A) John Jacob Westervelt

(B) her father

(C) a missionary

(D) a childhood friend

(E) a shipbuilder neighbor

18. In line 12, the word “restored” most nearly means

(A) updated

(B) refurbished

(C) put into storage

(D) deposited

(E) returned

1Prophetic

Excerpted from “The Story of a Pioneer” by Anna Howard Shaw, in Autobiographies of American Women: An Anthology © 1992 by Jill Ker Conway, ed., pp. 475–477

19. Which of the following best describes the relationship between the narrator and the men in her life?

(A) She gladly provides for their needs.

(B) She considers herself their equal.

(C) She feels overly dependent on them.

(D) She wishes to avoid them.

(E) She believes that they suppress her wishes.

20. The author was “emancipated” (line 34) so that she might more easily

(A) spend time with her father

(B) play with her young friends

(C) travel throughout New Bedford

(D) work with tools

(E) move to Michigan

21. In line 43, the word “movements” most nearly means

(A) travels

(B) cosmetic alterations

(C) cultural changes

(D) physical actions

(E) mechanical workings

22. The author indicates that she regarded New England as superior to Michigan in that New England

I. had humbler citizens

II. was more culturally developed

III. had finer gardens

(A) II only

(B) III only

(C) I and II only

(D) I and III only

(E) I, II, and III

23. The author”s attitude toward her move to Michigan is best described as

(A) eager

(B) awed

(C) fearful

(D) resentful

(E) bewildered

24. The sisters refused to ride in the lumber wagon mainly because

(A) they were embarrassed by its appearance

(B) they felt it was unsafe

(C) they had bad memories of it

(D) it was cold

(E) it lacked sufficient room for both of them

If you finish before time is called, you may check your work on this section only. Do not turn to any other section of the test.

SECTION 7
Time—20 minutes
16 questions


Turn to Section 7 of your answer sheet to answer the questions in this section.



Directions: For this section, solve each problem and decide which is the best of the choices given. Fill in the corresponding circle on the answer sheet. You may use any available space for scratchwork.



Notes

1. The use of a calculator is permitted.

2. All numbers used are real numbers.

3. Figures that accompany problems in this test are intended to provide information useful in solving the problems. They are drawn as accurately as possible EXCEPT when it is stated in a specific problem that the figure is not drawn to scale. All figures lie in a plane unless otherwise indicated.

4. Unless otherwise specified, the domain of any function f is assumed to be the set of all real numbers x for which f (x) is a real number.



Reference Information

The number of degrees of arc in a circle is 360.

The sum of the measures in degrees of the angles of a triangle is 180.


1. If , what is the value of ?

(A) 3

(B) 7

(C) 16

(D) 23

(E) 30

2. If one serving of cereal is cup, how many servings are in 3 pints of cereal? (1 pint = 2 cups)

(A) 3

(B) 9

(C) 18

(D) 27

(E) 36

3. If the radius of the circle with center O above is 4, what is the length of arc RST?

(A) 2π

(B) 4π

(C) 8π

(D) 12π

(E) 16π

4. In the triangle above, what is the value of x?

(A) 7

5. For , let ∇x be defined by the equation . Which of the following is equivalent to

(A) ∇2

(B) ∇3

(C) ∇6

(D) ∇8

(E) ∇9

6. Stephanie can clean a pool in 1 hour, and Mark can clean the same pool in 1.5 hours. If the rate at which they work together is the sum of their rates working separately, how many minutes should they need to clean the pool if they work together? (1 hour = 60 minutes)

(A) 24 minutes

(B) 36 minutes

(C) 60 minutes

(D) 72 minutes

(E) 100 minutes

7. Which of the following has the greatest value?

(A) (1003)4

(B) (1005)(1006)

(C) (10,000)4

(D) (1002 × 1002)2

(E) (1,000,000)3

8. Line m (not shown) is the reflection of line l over the x-axis. What is the slope of line m?

(A) 3/2

(B) 2/3

(C) 0

(D) –2/3

(E) –3/2

9. If and , what is the value of ?

(A) 10

(B) 12

(C) 14

(D) 16

(E) 18

10. The figure above shows the dimensions, in feet, of a stone slab. How many of these slabs are required to construct a rectangular patio 24 feet long and 12 feet wide?

(A) 18

(B) 20

(C) 24

(D) 36

(E) 48

11. $12,000 in winnings for a golf tournament were distributed in the ratio of 7:2:1 to the first-, second-, and third-place finishers, respectively. How much money did the first-place finisher receive?

(A) $1,200

(B) $1,700

(C) $2,400

(D) $8,400

(E) $10,000

12. If and, what is the value of ?

(A) 5

(B) 8

(C) 15

(D) 17

(E) 19

13. If r and s are positive integers and , which of the following must be true?

I. s is odd

II. r is even

is an integer

(A) I only

(B) III only

(C) I and II only

(D) I and III only

(E) I, II, and III

14. A bag contains six chips, numbered 1 through 6. If two chips are chosen at random without replacement and the values on those two chips are multiplied, what is the probability that this product will be greater than 20?

2,-4,-8,…

15. In the sequence above, each term after the second is equal to the product of the two preceding terms. For example, the third term, –8, is the product of 2 and –4. How many of the first 100 terms of this sequence are negative?

(A) 33

(B) 34

(C) 50

(D) 66

(E) 67

16. In the figure above, points C and D are midpoints of edges of a cube. A triangle is to be drawn with R and S as two of the vertices. Which of the following points should be the third vertex of the triangle if it is to have the largest possible perimeter?

(A) A

(B) B

(C) C

(D) D

(E) E

If you finish before time is called, you may check your work on this section only. Do not turn to any other section of the test.

SECTION 8
Time—20 minutes
19 questions


Turn to Section 8 of your answer sheet to answer the questions in this section.



Directions: For each question in this section, select the best answer from among the choices given and fill in the corresponding circle on the answer sheet.



Each sentence below has one or two blanks, each blank indicating that something has been omitted. Beneath the sentence are five words or sets of words labeled A through E. Choose the word or set of words that, when inserted in the sentence, best fits the meaning of the sentence as a whole.

EXAMPLE:

Rather than accepting the theory unquestioningly, Deborah regarded it with-----.

(A) mirth

(B) sadness

(C) responsibility

(D) ignorance

(E) skepticism


1. The latest review for the restaurant was -------, suggesting that the ------- cuisine came close to compensating for the insipid decor.

(A) glowing . . indefatigable

(B) banal . . mediocre

(C) ambivalent . . sublime

(D) severe . . piquant

(E) antiquated . . tepid

2. As unexpected as the results of the experiment were, Dr. Thompson refused to characterize them as -------.

(A) meticulous

(B) belligerent

(C) anomalous

(D) convergent

(E) warranted

3. The executives could only hope that the company”s poor first-quarter performance was not ------- of the year to come.

(A) an amalgam

(B) a harbinger

(C) an arbiter

(D) a deception

(E) a talisman

4. Around 1850, abolitionist and author Frederick Douglass sought to ------- those oppressed by slavery by facilitating the underground railroad, a widespread network of individuals and organizations that worked to transport former slaves out of bondage.

(A) evaluate

(B) encumber

(C) unfetter

(D) disorient

(E) forgo

5. Known for her ------- and decorative poetry, the author demonstrated her ------- by scribing a keenly analytical mystery novel.

(A) flamboyant . . immutability

(B) austere . . elegance

(C) unadorned . . flexibility

(D) florid . . versatility

(E) grandiloquent . . insurgence

6. Because the mechanisms by which cancers attack the body are so -------, scientists have been ------- in their efforts to find a universal cure.

(A) efficacious . . bilked

(B) multifarious . . stymied

(C) conspicuous . . thwarted

(D) consistent . . hampered

(E) lucid . . proscribed


The passages below are followed by questions based on their content; questions following a pair of related passages may also be based on the relationship between the paired passages. Answer the questions on the basis of what is stated or implied in the passage and in any introductory material that may be provided.


Questions 7–19 are based on the following passages.

The following passages are excerpts from a recent debate between two well-known astronomers. The author of Passage 1 is a professor of geological sciences and the author of Passage 2 is a principal scientist in the Department of Space Studies in Boulder, Colorado.

First passage: Peter Ward, “Great Debates Part I,” Astrobiology Magazine, 2003

Second passage: David Grinspoon, “Great Debates Part III,” Astrobiology Magazine, 2003

7. The discussion of the Drake equation in the first paragraph indicates that the author holds which of the following assumptions?

(A) The Drake equations are too complicated for most people to understand.

(B) Mathematical formulas can influence public opinion.

(C) Sagan did not substantially alter the Drake equation.

(D) Mathematics tend to obscure scientific exploration.

(E) Drake was not as reputable a scientist as Sagan was.

8. Which of the following best describes the function of the third paragraph?

(A) It asks more questions similar to those posed in the second paragraph.

(B) It provides more background information on the debate discussed in the passage.

(C) It explains a comment made in the second paragraph.

(D) It defines an important term mentioned in the second paragraph.

(E) It presents an opinion contrary to one presented in the second paragraph.

9. In line 46, the word “abiotic” most nearly means

(A) resistant to bacteria

(B) devoid of life

(C) highly populated

(D) extremely advanced

(E) quick growing

10. Which of the following best summarizes the main idea of Passage 1?

(A) The conditions that support complex life may be much more difficult to maintain than is widely assumed.

(B) The Drake equation is not a valid predictor of life in the universe.

(C) Evolution on Earth has made it very unlikely that there would be complex life on other planets.

(D) The number of planets in the universe with complex life is astronomical.

(E) Conditions allowing for the existence of microbes are rare.

11. In line 57, “grade” most nearly means

(A) level

(B) slope

(C) evaluation

(D) life

(E) quantity

12. The author of Passage 1 makes all of the following claims in support of his argument EXCEPT

(A) Complex life on Earth was due in part to haphazard events.

(B) Higher life forms sometimes face the likelihood of extinction due to catastrophic events.

(C) The Earth”s carbon-in-water example is probably not the only way for life to come into existence.

(D) Simple forms of life are far more common than highly evolved life forms.

(E) The evolution of life can be affected by the positions of planets around a star.

13. The “sample size of one” (line 78) refers to

(A) the Milky Way galaxy

(B) Drake and Sagan”s data

(C) the planet Earth

(D) the Sun of our solar system

(E) mass extinction events

14. The quotations in lines 101–105 serve to

(A) show how the author would respond to someone who disagrees with him

(B) illustrate an argument for why there is no life on neighboring planets

(C) explain a theory the author has disagreed with his entire career

(D) describe a conversation the author had with a colleague

(E) illustrate the author”s confusion about the origin of alternate life forms

15. The author includes the anecdote in lines 121–131 in order to

(A) compare his cat to the complex life forms in nearby galaxies

(B) give supporting evidence to the claim that life in the universe is unique to the Earth

(C) caution scientists about drawing premature conclusions from one specific occurrence

(D) mock scientists who believe that animals such as cats can live on other planets

(E) show the result of an evolutionary process

16. In saying that “Life is opportunistic” (lines 132–133), the author of Passage 2 suggests that

(A) only the most cunning animals survive

(B) evolution takes advantage of the unique features of many different environments

(C) humans will likely always be the dominant species on Earth

(D) the theory of evolution is probably wrong

(E) all life forms seek to dominate others

17. The author of Passage 2 suggests that the “complex creatures” discussed in lines 142–148 are likely to believe that

(A) technological advancements are critical to their survival

(B) life is unique to planet Earth

(C) there is no life on other planets

(D) life on all planets originates in the same manner

(E) carbon is essential to the creation of life

18. The author of Passage 1 would most likely respond to the statement in Passage 2 that “The biosphere … offer” (lines 133–135) by saying that

(A) our planet also offers many dangers to the biosphere

(B) the biosphere is filled with far more complex life forms

(C) life on Earth has not evolved to such a high level

(D) our planet does not offer so many idiosyncrasies

(E) carbon is one of the most complex elements in the universe

19. The authors of both passages would most likely agree with which of the following statements?

(A) The estimates made by the Drake Equation are surprisingly accurate.

(B) Mass extinction events are not a factor in predicting the existence of extraterrestrial life.

(C) Mathematical models are the most helpful means of learning about the development of life in the universe.

(D) There is likely an abundance of life in the universe that has yet to be discovered.

(E) Complex life is very common in the universe.

If you finish before time is called, you may check your work on this section only. Do not turn to any other section of the test.

SECTION 9
Time—10 minutes
14 questions


Turn to Section 9 of your answer sheet to answer the questions in this section.



Directions: For each question in this section, select the best answer from among the choices given and fill in the corresponding circle on the answer sheet.



The following sentences test correctness and effectiveness of expression. Part of each sentence or the entire sentence is underlined; beneath each sentence are five ways of phrasing the underlined material. Choice A repeats the original phrasing; the other four choices are different. If you think the original phrasing produces a better sentence than any of the alternatives, select choice A; if not, select one of the other choices.

In making your selection, follow the requirements of standard written English; that is, pay attention to grammar, choice of words, sentence construction, and punctuation. Your selection should result in the most effective sentence—clear and precise, without awkwardness or ambiguity.

EXAMPLE:

The children couldn”t hardly believe their eyes.

(A) couldn”t hardly believe their eyes

(B) could hardly believe their eyes

(C) would not hardly believe their eyes

(D) couldn”t nearly believe their eyes

(E) couldn”t hardly believe his or her eyes


1. His morning routine included eating an English muffin with grape jelly, then to drink coffee from a styrofoam cup, and sitting down to draw his daily comic strip.

(A) then to drink coffee from a styrofoam cup

(B) drinking coffee from a styrofoam cup

(C) then drink coffee from a styrofoam cup

(D) from a styrofoam cup he would drink coffee

(E) he would drink coffee from a styrofoam cup

2. Pretending to have hurt his knee, Mark”s attempt to convince his coach to let him out of practice was a failure.

(A) Mark”s attempt to convince his coach to let him out of practice was a failure

(B) Mark”s attempt to convince his coach failed to let him out of practice

(C) Mark attempted to convince his coach to let him out of practice, but it was a failure

(D) Mark attempted to convince his coach to let him out of practice, but failed

(E) Mark attempted to convince his coach in letting him out of practice, but failed

3. The flier describing the details of the blood drive requested that we are in the hospital lobby promptly at 10 A.M.

(A) are in the hospital lobby

(B) should get at the hospital lobby

(C) be in the hospital lobby

(D) would be to the hospital lobby

(E) should have been at the lobby of the hospital

4.Known for his temper, impatience, and how easily he can be irritated, Dr. McGee was not well liked by his patients.

(A) Known for his temper, impatience, and how easily he can be irritated

(B) Knowing his temper, impatience, and irritability

(C) Known for his temper, impatience, and irritability

(D) Known for his temper, impatience, and irritation

(E) Known for his temper, for his impatience, and his irritability

5.Winning the final match, Courtney gave a gracious speech thanking her competitor, the sponsors, and the spectators.

(A) Winning

(B) Having won

(C) Being that she won

(D) If she had won

(E) For her winning

6. Generally regarded as the most influential social science treatise of the 20th century, John Maynard Keynes wrote a book, The General Theory of Employment Interest and Money that forever changed the way scientists looked at the economy.

(A) John Maynard Keynes wrote a book, The General Theory of Employment Interest and Money that

(B) a book by John Maynard Keynes, The General Theory of Employment Interest and Money, that

(C) John Maynard Keynes” book The General Theory of Employment Interest and Money had already

(D) John Maynard Keynes wrote a book The General Theory of Employment Interest and Money having

(E) John Maynard Keynes” book The General Theory of Employment Interest and Money

7.Neither of the proposals remained in their original form by the time the legislature finished its deliberations.

(A) Neither of the proposals remained in their original form

(B) Neither proposal remained in its original form

(C) Both of the proposals did not remain in its original form

(D) With neither proposal remaining in its original form

(E) Neither proposal remained in their original forms

8. The Chief of Staff worked through the night to prepare the President”s speech for the following day.

(A) to prepare

(B) in preparing

(C) in the preparation of

(D) for preparing

(E) in order for preparing

9. The storm waves crashed into the shore, inundating the stores along the boardwalk and many cars in the parking lot were swept away by them.

(A) boardwalk and many cars in the parking lot were swept away by them

(B) boardwalk with many cars in the parking lot being swept away

(C) boardwalk and sweeping away many cars in the parking lot

(D) boardwalk, and it swept away many cars in the parking lot

(E) boardwalk; sweeping away many cars in the parking lot

10. The life of the ShinZanu, a tribe of the Australian Outback, have been realistically depicted in the books of Ronald Skinner.

(A) The life of the ShinZanu, a tribe of the Australian Outback, have been realistically depicted in the books of Ronald Skinner.

(B) The life of the ShinZanu tribe of the Australian Outback has been realistically depicted in the books of Ronald Skinner.

(C) The ShinZanu, a tribe of the Australian Outback, has had its life realistically depicted with the books of Ronald Skinner.

(D) Ronald Skinner has depicted the life of the ShinZanu realistically in his books; they are of the Australian Outback.

(E) Depicting the lives of the ShinZanu tribe of the Australian Outback realistically, Ronald Skinner has done that in his books.

11. At the age of seven, my father took me to see my first baseball game.

(A) At the age of seven, my father took me to see

(B) My father took me, at the age of seven, to see

(C) Being seven years old, my father took me to see

(D) When I was seven years old, my father took me to see

(E) I was taken by my father at seven years old, seeing

12. The President worked hard to implement legislation that would stimulate growth, curb inflation, and increase employment.

(A) that would stimulate growth, curb inflation, and increase employment

(B) stimulating growth, curbing inflation, and to increase employment

(C) that stimulated growth, curbed inflation, and increasing employment

(D) to stimulate growth, the curbing of inflation, and increasing employment

(E) in order to stimulate growth, and for the purpose of curbing inflation and increasing employment

13. If anyone asks for a doctor, send them directly to the nurses” station for immediate assistance.

(A) If anyone asks for a doctor, send them

(B) Having asked for a doctor, send them

(C) When anyone asks for a doctor, they should be sent

(D) Had anyone asked for a doctor, send them

(E) Send anyone who asks for a doctor

14. Even if they have been declawed as kittens, adult cats often run their paws along tall objects as if to sharpen their claws.

(A) Even if they have been declawed as kittens

(B) Even though they should have been declawed when being kittens

(C) Even when being declawed as kittens

(D) Declawed when kittens nevertheless

(E) Declawed as kittens

If you finish before time is called, you may check your work on this section only. Do not turn to any other section of the test.

ANSWER KEY

SCORE CONVERSION TABLE

How to score your test

Use the answer key on the previous page to determine your raw score on each section. Your raw score on each section except Section 5 is simply the number of correct answers minus ¼ of the number of wrong answers. On Section 5, your raw score is the sum of the number of correct answers for questions 1–18 minus ¼ of the number of wrong answers in questions 1–8. Next, add the raw scores from Sections 3, 6, and 8 to get your Critical Reading raw score, add the raw scores from Sections 2, 5, and 7 to get your Math raw score, and add the raw scores from Sections 4 and 9 to get your Writing raw score. Write the three raw scores here:

Raw Critical Reading score: ____________ Raw Math score: ____________ Raw Writing score: ___________

Use the table below to convert these to scaled scores.

Scaled scores: Critical Reading: _____________ Math: _____________ Writing: _____________

SCORE CONVERSION TABLE FOR WRITING COMPOSITE
[ESSAY + MULTIPLE CHOICE]

Calculate your Writing raw score as you did on the previous page and grade your essay from a 1 to a 6 according to the standards that follow in the detailed answer key.

Essay score: ____________ Raw Writing score: ____________

Use the table below to convert these to scaled scores.

Scaled score: Writing: _____________

Detailed Answer Key

Section 1


Consider carefully the issue discussed in the following passage, then write an essay that answers the question posed in the assignment.


Many among us like to blame violence and immorality in the media for a “decline in morals” in society. Yet these people seem to have lost touch with logic. Any objective examination shows that our society is far less violent or exploitative than virtually any society in the past. Early humans murdered and enslaved each other with astonishing regularity, without the help of gangsta rap or Jerry Bruckheimer films.


Assignment: Do violence and immorality in the media make our society more dangerous and immoral? Write an essay in which you answer this question and discuss your point of view on this issue. Support your position logically with examples from literature, the arts, history, politics, science and technology, current events, or your experience or observation.



The following essay received 6 points out of a possible 6. This means that, according to the graders, it

• develops an insightful point of view on the topic

• demonstrates exemplary critical thinking

• uses effective examples, reasons, and other evidence to support its thesis

• is consistently focused, coherent, and well organized

• demonstrates skillful and effective use of language and sentence structure

• is largely (but not necessarily completely) free of grammatical and usage errors


One of the most misguided notions of conventional wisdom is that depicting violence in the media makes our society more violent. A close examination shows that this claim is baseless. Societies with severe restrictions on violence in the media tend to be more, not less, violent than those with no such restrictions. Indeed, despite the popular myth of a more peaceful past, societies were far more violent before the advent of movies, television, and video games. Societies that restrict access to “immoral” western movies are the same ones that call their citizens to violent and irrational holy war.

As Michael Moore pointed out poignantly in the movie “Bowling for Columbine,” Americans kill each other with firearms at a far greater rate than almost any other first-world nation. But he is quick to point out that our media is not more violent than those in Japan or Germany or even Canada, which have rates of violence that are a full order of magnitude lower than ours. Indeed, the killers among us are not likely to spend a lot of time listening to Marilyn Manson or playing Mortal Kombat on their Playstations, despite what our more nearsighted and sanctimonious politicians and preachers would like us to believe. Ted Kaczynski, the Unabomber, lived in a one-room shack without electricity or running water, let alone cable. But even if murderers like Kaczynski were video game addicts, attributing their motives to media violence would be missing the point entirely.

People who are habitually violent have adopted a “war mentality.” They tend to see the world in black-and-white, us-against-them terms. Tragically, our leaders tend to have this very same mentality, but they couch it in “patriotism.” Lobbing cruise missiles and landing marines in another country is not considered a horrible last resort, but a patriotic duty. If we wish to understand why Americans are more violent than the Japanese, violence in the media will hold no answers; Japanese kids watch just as much violence. Foreign policy is far more telling: which country has leaders who engage in violence against other countries at every opportunity, and constantly try to convince us that it”s right?

If our pundits and politicians were truly concerned about making a safer world—and there are many reasons to believe they are not, since they profit the most from a fearful citizenry—they would begin by acknowledging that violence is almost a desperate grab for control from a person or people who believe they are being repressed. If we want a more peaceful and noble society, then we will stop coercing other countries with violence and economic oppression. As Franklin Roosevelt said, “We have nothing to fear but fear itself.” We are the most fearful nation on the planet, and we are paying for it.


The following essay received 4 points out of a possible 6, meaning that it demonstrates adequate competence in that it

• develops a point of view on the topic

• demonstrates some critical thinking, but perhaps not consistently

• uses some examples, reasons, and other evidence to support its thesis, but perhaps not adequately

• shows a general organization and focus, but shows occasional lapses in this regard

• demonstrates adequate but occasionally inconsistent facility with language

• contains occasional errors in grammar, usage, and mechanics


People say that society today is much more violent due to all of the media portrayal of violence we see on a daily basis. The nightly news is often made up entirely of stories about murders, muggings, arson, and other gruesome crimes. The most successful shows on television are the investigative crime shows in which they solve disturbing murder mysteries. Movies like the Lord of the Rings contain gory fight scenes that show the death of hundreds of characters. It”s hard even to find a video game anymore that doesn”t somehow relate back to fighting.

Those who don”t believe that violence breeds violence would argue that the United States murder rate had declined to its lowest level in 30 years and that this is proof that the violence in the media has not in fact made for a more violent society. But what they conveniently leave out is the fact that at the same time, youth gun killings were on the rise. This is who is being affected by the increased exposure to violence—the children. It is perhaps the video game violence and television/movie violence that can be held responsible.

Kids today are growing up in a society where violence is everywhere. It is difficult for a child to go through the day without witnessing some violent act on TV or hearing about a gruesome murder on the radio. A recent study we learned about in class concluded that because of what they see on television, children become immune to violence, accept it as something that is part of a “normal” life, and they often times will attempt to imitate what they see on television because it “looks fun.”

Something needs to be done to reverse this trend of growing violence in our country and tighter regulation of the amount of violence on television, in music, and in the movies would be a great place to start. The youth of this country need to be reminded that violence is not an acceptable part of daily existence and that it should be avoided at all costs.


The following essay received 2 points out of a possible 6, meaning that it demonstrates some incompetence in that it

• has a seriously limited point of view

• demonstrates weak critical thinking

• uses inappropriate or insufficient examples, reasons, and other evidence to support its thesis

• is poorly focused and organized and has serious problems with coherence

• demonstrates frequent problems with language and sentence structure

• contains errors in grammar and usage that seriously obscure the author”s meaning


Believing that the violence in the media has made the members of our society like violent murderers is an absurd notion. Sure, there are lots video games on the market that involve fighting ninjas and battling army troops. Yes, nightly television shows on the public television networks show many a violent episode. Sure, the nightly news is covered with violent crimes and such. For instance, the popular music of this era is full of violent references and foul language. But, no experiment or statistics that I have seen proves the above statement to be true. Just because a teenager kills over 500 fake people on his ninja fighting videogame, it does not mean that after he turns off the game console that he will run outside in his ninja costume and start attacking the people in his neighborhood. It is absurd to say that violence is because of all the violence on video games television. Actually I think that video games make you better at eye-hand coordination which is a valuable skill. Hundreds of years before video games and movies and television, there were murder and violence. Human beings are violent people and the exposure to violence does not make us more violent than we already were. If we did not have all of these impressive technological advances such as radio, television and film, we would still be committing acts of violence. There will always be violent humans that are ready to hurt others to get what they want and eliminating violent references from our music and television shows might even make people madder.

Section 2

(Chapter 8, Lesson 1: Solving Equations)

2. B Write out a mathematical equation for how you would actually find the cost for the month: $.95 × 31. Answer choice B, $1.00 × 30, is closest to that amount.

(Chapter 7, Lesson 1: Numbers and Operations)

3. D A linear angle measures 180°. Write an equation:

(Chapter 10, Lesson 1: Lines and Angles)

(Chapter 11, Lesson 2: Functions)

5. D The difference between x and y is (x-y).

The sum of x and y is (x + y).

The product of those two is equal to 18:

(Chapter 8, Lesson 5: Factoring)

(Chapter 8, Lesson 4: Working with Roots)

7. C Let b = cost of chocolate bar and g = cost of gum.

(Chapter 8, Lesson 1: Solving Equations)

8. C First find 40% of 80:
Now find what percent of 96 is 32.

(Chapter 7, Lesson 5: Percents)

9. A If and both land mare integers, then m must be either 1, 3, 7, or 21. If , however, then m must also be a factor of 39, so it must be 3. Therefore, and , so .

(Chapter 8, Lesson 6: Inequalities, Absolute Values, and Plugging In)

10. D There”s no need to do a lot of calculation here. Look for the two adjacent bars with the greatest positive difference between them. Since 1999 shows the least profits of all the years on the graph and 2000 shows the greatest profits of any year on the graph, 1999–2000 must have the greatest change in profit.

(Chapter 11, Lesson 5: Data Analysis)

11. B A Venn diagram can help you with this problem: Imagine that the 4 students who play two sports play soccer and tennis. (It doesn”t matter which specific pair of sports they play.) This means that students play just soccer, students play just tennis, and 9 students play just lacrosse. This shows that there is a total of students.

(Chapter 9, Lesson 5: Counting Problems)

12. B To solve this problem, you need to find the distance between the center of the circle (14, 14) and the point on the circle (2, 9). To do this, you can use the distance formula.

You can also draw a right triangle connecting the two points. It gives you a triangle with one leg of 5 and one leg of 12. Set up the Pythagorean theorem and solve for r.

(Chapter 10, Lesson 3: The Pythagorean Theorem)

13. B The population doubles every 18 months. Start with January of 2000 and start doubling.

(Chapter 9, Lesson 3: Numerical Reasoning Problems)

14. C Use the Fundamental Counting Principle from Chapter 9, Lesson 5. To arrange these students, five choices must be made. First select the students for each end. Since one of the five (the tallest) cannot go on either end, you have four students to choose from for one end, and then, once that choice has been made, three students to choose from for the other end:

Now fill the remaining spots. There are three students left to choose from for the second spot:

Then, once that selection has been made, there are two for the next spot, then one for the remaining spot:

To find the total number of possible arrangements, simply multiply:.

(Chapter 9, Lesson 5: Counting Problems)

15. B From the diagram, we know that , and we know that .

If you want b to be as large as possible, then you need to make the sum of a and c as small as possible. The smallest integer value of a possible is 91. So let”s say that a = 91.

So 43 is the largest possible value of c; this means that is the largest possible value of b.

(Chapter 10, Lesson 2: Triangles)

16. B Begin by finding the area of the big equilateral triangle. An equilateral triangle with sides of length 4 has a height of because the height divides the triangle into two 30°-60°-90° triangles.

The big triangle is divided into four equal parts, three of which are shaded, so the shaded area is ¾ of the total area.

(Chapter 10, Lesson 5: Areas and Perimeters)

17. D Just look at the graph and draw a line at .

The y-values of the graph are at or above that line from to and from to .

(Chapter 11, Lesson 2: Functions)

18. C This table shows all of the possible values of ab:

Of those, only the seven shaded values are greater than 20 and less than 50, so the probability is 7/25.

(Chapter 9, Lesson 6: Probability Problems)

(Chapter 8, Lesson 3: Working with Exponentials)

20. D The graph of is the graph of shifted to the right two units without changing its shape. Therefore, the “peak” at point (6, 4) should shift to (8, 4).

(Chapter 11, Lesson 3: Transformations)

Section 3

1. A The word although indicates a contrast. Raúl purchased his computer only 10 months ago, but technology has been improving so fast that it is already outdated. obsolete = outdated; adjunct = auxiliary, or additional; novel = new, innovative; elusive = hard to catch

2. C The admissions committee is looking to justify offering more scholarships to increase the number of applications, so the number of applicants must be decreasing. mushroom = expand rapidly; plummet = decrease rapidly; satiate = satisfy; burgeon = grow

3. B If the father will not consider another person”s viewpoint to be valid if it differs from his own, he must be pretty stubborn or arrogant. pragmatic = practical; dogmatic = arrogantly authoritative; phlegmatic = sluggish; cordial = polite; curt = abrupt and rude

4. E The books are written for children but are still enjoyable to adults. penned = written; prosaic = dull; morose = gloomy; censored = cleansed of profanity; incongruous = not compatible; tedious = boring, dull; authored = written; engaging = captivating, interesting

5. E Julia is at the top of her class, but if this is hard to believe, she must approach her work in a lazy or irresponsible way. adept = skilled; diligent = hardworking; fanatical = obsessive and crazy; extroverted = outgoing; laggardly = slow-moving, lagging behind

6. A The President”s opponents were always cautious about debating him, so the President must be highly skilled or intimidating or mean. redoubtable = formidable, imposing; staid = calm, not outwardly emotional; magnanimous = generous; stoic = indifferent to pain or pleasure

7. D The new clothing line was described as being eclectic (containing much variety). It ranged from modest (not showy) and unadorned (undecorated) to -------- and garish (flashy). By parallelism, the missing word should be in opposition to the word modest. austere = severe, stern;prophetic = able to tell the future; cordial = polite; ostentatious = showy; solitary = alone

8. C The textbook includes all of the essential information but it is not verbose (wordy); the two missing words should be parallel to containing lots of information and not verbose. compendious = succinct; circumlocutory = talking around the subject, indirect; reprehensible = blameworthy;terse = concise; comprehensive = including a large amount of information; concise = brief and to the point; grandiloquent = speaking in a pompous manner; painstaking = done with great care; redundant = repetitive

9. B Saying that we were raised in unrivaled prosperity is like saying that the economy has been very strong and abundant.

10. E The “people” are plagued by deep divisions (line 9), and the citizens are the only ones who are not growing to appreciate the difference between America and the United States (lines 20–22). Therefore, the people lack unity, while the citizens lack awareness.

11. A Don”t miss the word EXCEPT in the question. Choice (B) is supported in line 14, choice (C) in line 7, choice (D) in line 8, and choice (E) in line 8. The last lines say that ambition for a better life is now universal, implying that not everyone is happy with the status of their lives.

12. A Unlike Passage 1, Passage 2 discusses the difference between the ideal of America and the reality of the United States.

13. A The questions in the opening lines show the man”s confusion, and the woman is said to talk ardently (passionately).

14. D The author says that one who is suddenly overwhelmed by terror cannot afterwards remember the exact order of sounds accompanying the catastrophe which stuns him—that is, he becomes disoriented.

15. C Line 11 suggests that Ognev is stunned by a catastrophe. The context of the passage makes it clear that this catastrophe is the expression of love from Vera, which Ognev has difficulty understanding.

16. D In saying that she had been struck by … the aims and objects of his life (lines 22–24), the author is saying that she was impressed with Ognev”s life goals.

17. E In lines 42–44, the passage states that much as he wanted to, he could feel no joy; no fundamental happiness. In other words, the bad and strange thing was disaffection.

18. E In lines 51–52, the passage states that Vera”s raptures and suffering seemed to him (Ognev) to be only cloying (excessively sweet) and trivial (of little significance). He felt her passion to be unimportant and was outraged at himself for feeling this way. To him, his statistics, books or philosophical truths were more important than this passion.

19. A The final sentence of the passage states that he was annoyed and blamed himself even though he himself did not understand why he was to blame. Ognev is confused and uncertain about how he should feel about Vera”s passion. He feels indifference but thinks he should feel something different.

20. D In lines 21–22 the marchers are described as singing to hide their exhaustion and then as trying not to fear free-thinking This commitment to hiding emotion is stoicism.

21. A Lines 35–38 criticize the bill”s failure to protect the right of African Americans to vote “when local officials are determined to deny it.” In other words, it did not sufficiently pressure local officials to extend voting rights to all citizens.

22. B In context, saying that his … encounter with Mexican-American children was shattering is like saying that the encounter bothered the President and had a major impact on the way he approached civil rights issues later in his career.

23. C Johnson indicates that he inferred, by looking into his students” eyes, that they knew that others disliked them. This indicates a strong empathy with his students, because he inferred it not from their words but from their expressions.

24. E Lines 52–54 say that Johnson made the nationwide audience aware of how deeply personal the issue of African American rights was to him and lines 60–62 say that he spoke more directly, more explicitly, and more warmly of the human experience of prejudice than any president before him. In other words, he addressed it directly and in personal terms.

Section 4

1. C The word group is the singular subject, so the verb should be was.(Chapter 15, Lesson 1: Subject-Verb Disagreement)

2. A The original sentence is best. Choice (B) is incorrect because the phrase of labor”s and management”s is redundant and unidiomatic. In choice (C), the use of the verb saw is non-standard, although idiomatic, and the colon is misused because it is not followed by a list or an explanatory independent clause. In choice (D), the phrase marked ferociously is illogical. In choice (E), both the tense and voice of the verb, was marking, are illogical.

3. D The original sentence is awkward, and its verb has possessed does not agree with the plural subject, scientists and inventors. Choice (B) is incorrect because the verb have been able to possess is not logical. Choices (C) and (E) are incorrect because the verb was does not agree with the plural subject skills. (Chapter 15, Lesson 1: Subject-Verb Disagreement)

4. B In the original sentence, the pronoun it lacks a clear antecedent, as does the pronoun that in choice (D). Choice (C) is incorrect because it implies that the Thracians enabled the ability, which is illogical. In choice (E), the phrase ability of resisting is unidiomatic. (Chapter 15, Lesson 5: Pronoun-Antecedent Disagreement)

5. C The phrase on an overseas journey is redundant because the sentence also states that this journey was across the Atlantic. This redundancy is repeated in choice (B). Choice (D) uses an incorrect verb tense, and choice (E) produces a sentence fragment. (Chapter 15, Lesson 12: Other Modifier Problems)

6. E In the original sentence, the use of plus instead of and is non-standard, and the phrasing is not parallel. Only choice (E) avoids both problems. (Chapter 15, Lesson 3: Parallelism)

7. C The phrase half as many … than is unidiomatic. The correct idiom is half as many as. Only choice (C) is phrased idiomatically. (Chapter 15, Lesson 10: Idiom Errors)

8. D The use of therefore in the original phrasing is illogical, because the ideas in the sentence are related not as a cause and effect but rather as a contrast. The use of actually in choice (D) conveys the appropriate irony. (Chapter 15, Lesson 15: Coordinating Ideas) (Chapter 12, Lesson 7: Write Logically)

9. C In the original sentence, the prepositional phrase for their fans is unidiomatic and awkward, and the pronoun their does not agree in number with its antecedent, the band. Choices (B) and (D) repeat the pronoun problem. Choice (E) is incorrect because it implies that the fans are free of charge, rather than the downloading. (Chapter 15, Lesson 5: Pronoun-Antecedent Disagreement)

10. C The plural subject reasons requires the plural verb are, so choices (A) and (D) are incorrect. Choice (B) is incorrect because the phrase the reasons are because is non-standard and illogical. Choice (E) is incorrect because the phrasing is not parallel. (Chapter 15, Lesson 1: Subject-Verb Disagreement)

11. B The appositive phrase An untiring defender of the downtrodden must be placed adjacent to the noun it modifies, which in this case is Clarence Darrow. Only choices (B) and (E) do this, but choice (E) is incorrect because it lacks parallel phrasing. (Chapter 15, Lesson 8: Other Misplaced Modifiers)

12. D The subject of this verb is delivery, which is singular, so the verb should be has been.(Chapter 15, Lesson 1: Subject-Verb Disagreement)

13. C The phrase capable to distinguish is unidiomatic. The correct phrasing is capable of distinguishing. (Chapter 15, Lesson 10: Idiom Errors)

14. E The sentence is correct.

15. B The subject of this verb is photographs … and diagrams, which is plural, so the verb should be were.(Chapter 15, Lesson 1: Subject-Verb Disagreement)

16. D This is a comparison error. The way in which chimpanzees form friendships cannot logically be compared to humans. Instead, the phrase should be to the way humans form friendships.(Chapter 15, Lesson 4: Comparison Problems)

17. B As a noun, affects means feelings or emotions, so its use here is a diction error. The proper word is effects.(Chapter 15, Lesson 11: Diction Errors)

18. B There are two errors in this phrase. First, the subject probability is singular, so the verb should be is. Second, a probability can be lower than another, but not fewer than another. (Chapter 15, Lesson 1: Subject-Verb Disagreement) (Chapter 15, Lesson 4: Comparison Problems)

19. C The pronoun they does not agree in number with its antecedent an author, and should be replaced with the phrase he or she. (Chapter 15, Lesson 5: Pronoun-Antecedent Disagreement)

20. D The phrase in the time is redundant because the word during conveys the same information. The entire phrase should be deleted. (Chapter 15, Lesson 12: Other Modifier Problems)

21. C People are satisfied with things, not at them. (Chapter 15, Lesson 10: Idiom Errors)

22. A The phrase had ate is an incorrect past perfect form. The correct form is had eaten. In this case, however, the word after conveys the time sequence, so the past perfect form isn”t strictly necessary: ate (but not had ate) is an acceptable alternative. (Chapter 15, Lesson 9: Tricky Tenses)

23. E The sentence is correct as written.

24. D The verb is does not agree in number with its plural subject, jaws, and should be changed to are. (Chapter 15, Lesson 1: Subject-Verb Disagreement)

25. C The subject of the verb help is taking, which is singular. Think of the subject as it. The word help should instead be helps.(Chapter 15, Lesson 1: Subject-Verb Disagreement)

26. B The word underneath means physically below something. The word should instead be under.(Chapter 15, Lesson 10: Idiom Errors)

27. C The subject they is referring to the company, which is singular. They should instead be it.(Chapter 15, Lesson 1: Subject-Verb Disagreement)

28. D The original phrasing is not parallel. When using the idiomatic phrase not only A but also B, the phrasing must be precise and the phrases in A and B must be parallel. Therefore, the phrase he or she should in choice D must be eliminated. (Chapter 15, Lesson 3: Parallelism)

29. D The comparison requires the idiomatic phrase more A than B. Therefore, the phrase and not should be replaced with than. (Chapter 15, Lesson 4: Comparison Problems)

30. B This phrasing is the most concise and logical of the choices. (Chapter 12, Lesson 7: Write Logically)

31. A The original phrasing is best.

32. D Because the sentence refers to these passions, it is most logically placed after those passions are described. It also provides a logical transition to the third paragraph. (Chapter 12, Lesson 7: Write Logically)

33. B This order places the sentences in proper logical and chronological order: (8) identifies his childhood passion, (10) identifies his goals for this passion, (7) proceeds to his college years, (6) mentions where he pursued his passions, and (9) describes the connection between these passions and his later career. (Chapter 12, Lesson 7: Write Logically)

34. C The paragraph as a whole discusses Roosevelt”s passion for nature, so details about his activities in these natural settings would be relevant. (Chapter 12, Lesson 7: Write Logically)

35. E This sentence would be a good conclusion to the passage because it gives historical perspective to the specific ideas in the passage. (Chapter 12, Lesson 12: Finish with a Bang)

Section 5

1. B There are 180° on the side of a line.

(Chapter 10, Lesson 1: Lines and Angles)

2. B The equation states that some number, when squared, equals 36. That number can be either 6 or –6. Taking the square root of both sides of the equation gives:

Therefore, the answer is (B)–2.

(Chapter 8, Lesson 1: Solving Equations)

3. C There are 180° in a triangle. Set up equations for the two triangles in the figure.

(Chapter 10, Lesson 2: Triangles)

(Chapter 11, Lesson 2: Functions)

5. C The ratio of the nuts is a part-to-part-to-part-to-part ratio. Adding these numbers gives the total number of parts:. Since four of these parts are almonds, the fraction of the mixture that is almonds is 4/18, or 2/9.

(Chapter 7, Lesson 4: Ratios and Proportions)

6. D If 20 students scored an average of 75 points, then the sum of their scores is total points.
If 12 of those students scored an average of 83 points, then the sum of their scores is points.
Therefore, the remaining 8 students scored points altogether, so their average score is points.

(Chapter 9, Lesson 2: Mean/Median/Mode Problems)

7. A The sides of square EFGH all have length A diagonal of this square can be found with the Pythagorean theorem:

(Or, more simply, you can remember that the length of the diagonal of a 45°-45°-90° triangle is the length of the side times So the diagonal is ) By the same reasoning, since the sides of square ABCD all have length

Notice that ; therefore,, so . By the same reasoning,, so .

(Chapter 10, Lesson 3: The Pythagorean Theorem)

8. D Although you were probably taught to add the “rightmost” digits first, here the “leftmost” digits provide more information about the number, so it”s best to start there.

The largest possible 3-digit number that can be formed by adding two 2-digit numbers is . Therefore, T must be 1.

Therefore, there must be a “carry” of 1 from the addition of R + S in the 10s column. Looking at the units column tells us that S + R yields a units digit of 4, so . The addition in the 10s column tells us that . (The “+ 10” is needed for the carry into the 100s column.)

(Chapter 9, Lesson 3: Numerical Reasoning Problems)

(Chapter 9, Lesson 1: New Symbol or Term Problems)

10. 750 25% of $600 is $150. Therefore, the club earned $150 more in 2007 than it did in 2006, or $600 + . Remember, also, that increasing any quantity by 25% is the same as multiplying that quantity by 1.25.

(Chapter 7, Lesson 5: Percents)

(Chapter 8, Lesson 2: Systems)

12. 32 Let , and let . Since x is twice the length of y,.

To find the area of the shaded region, you might notice that if PM is the base of the shaded triangle, then LO is the height, so .

If you don”t notice this, you can find the shaded area by finding the area of the rectangle and subtracting the areas of the two unshaded triangles.

Area of rectangle = (length)(width)
Area of

Area of triangle
Area of triangle

Area of triangle
Area of triangle

Area of triangle

(Chapter 10, Lesson 5: Areas and Perimeters)

(Chapter 8, Lesson 3: Working with Exponentials)

14. 3 Draw a line with points P, Q, R, andS on the line in that order. You are given that and that , so choose values for those lengths, like ,, and .

This means that , so .

(Chapter 6, Lesson 2: Analyzing Problems)

15. 1.5 Since the graph is a parabola, it has a vertical axis of symmetry through the vertex. The points (–1, 6) and (4, 6) have the same y-coordinate, so each one is the reflection of the other over the axis of symmetry. This axis, therefore, must be halfway between the two points. Since the average of –1 and 4 is , the axis of symmetry must be the line , and therefore .

(Chapter 11, Lesson 2: Functions)

16. 18 Since these numbers are “evenly spaced,” their mean (average) is equal to their median (middle number). The average is easy to calculate: . Therefore, the middle number is 22, so the numbers are 18, 20, 22, 24, and 26.

Alternatively, you can set up an equation to find the sum of five consecutive unknown even integers, where x is the least of these:

So the five integers are 18, 20, 22, 24, and 26.

(Chapter 9, Lesson 2: Mean/Median/Mode Problems)

17. 20 Use the percent change formula:

(Chapter 7, Lesson 5: Percents)

18. 25 Let b = the number of black marbles, w = the number of white marbles, and r = the number of red marbles in the jar. If you are four times as likely to choose a black marble as a white one, then . If you are five times as likely to choose a red marble as a black one, then . To find the least possible number of marbles in the jar, imagine you have only one white marble. This would mean you have black marbles and red marbles, for a total of marbles.

In general, you can represent the total number of marbles as

In other words, the number of marbles in the jar must be a multiple of 25. The smallest positive multiple of 25 is, of course, 25.

(Chapter 9, Lesson 6: Probability Problems)

Section 6

1. A If the fight did not ensue, John must have intervened to stop it. intervene = get in the way of something; coalesce = fuse together; intermingle = mix together; exacerbate = make worse

2. D The defendant hoped the testimony would corroborate (support) his alibi, which would clear him of blame. convoke = call together; synthesize = generate; absolve = free of blame; impeach = accuse

3. E Being ensnarled (tied up) in traffic is an unpleasant experience that Rachel would have an aversion to or dislike for. antipathy = feeling against; penchant = liking; predilection = liking; proclivity = tendency to do something; aversion = feeling of dislike; insufferable = intolerable

4. A If the practices are no longer considered state of the art, they must now be considered outdated or unsophisticated. The physicians are incredulous (not able to believe) that such barbaric acts were once supported or condoned. primitive = old, unsophisticated; sanctioned = approved;ingenious = incredible, brilliant; boorish = rude, censured = publicly condemned; innovative = new; endorsed = supported; foolhardy = recklessly bold; condemned = criticized

5. B The Prime Minister had vetoed the law in the past many times, so he didn”t want it to pass. What would come as a great surprise? The Prime Minister”s suddenly supporting the law. articulated = expressed clearly; championed = defended; denounced = spoke out against; initiated = began; abbreviated = shortened

6. C Lines 3–4 state that the tradition is that a man never lifts his hand against a woman. Furthermore, if a man offends a woman, she is entitled to give him a sound thrashing (line 6). Therefore, a man who disrespected a woman would face censure.

7. E Saying that it is not an unusual thing for a squaw to administer a sound thrashing to a warrior husband (lines 5–7) is like saying that it is not unusual for her to give him a beating, or dispense it.

8. C Lines 5–6 say that merely receiving palliative care … provides no hope of a cure. Therefore, palliative care only reduces the discomfort of the symptoms, without curing the disease, as something analgesic does.

9. A Lines 8–11 ask, How can a doctor know if a patient has the mental capacity to decide for herself that the time has come to stop fighting the disease? This question indicates that there may be some difficulty in determining a patient”s state of mind.

10. B The first sentence of the passage says there was great optimism about earthquake prediction. Each paragraph discusses potential precursors, or predictors, of earthquakes.

11. E Lines 8–10 say that because foreshocks look just like any other earthquake, they are not in themselves very useful in prediction.

12. D Support for choice II can be found in line 19, which says that ground-water has become cloudy prior to an earthquake. Choice III is supported in lines 16–18, which say that before a large earthquake, marked changes have been reported in the level or flow of wells and springs. Nothing is said about density changes in the ground-water.

13. A The passage says (lines 8–10) that since foreshocks look just like any other earthquakes, they are not in themselves very useful in prediction but later (lines 39–42) mentions that because the Haicheng earthquake had hundreds of foreshocks, it was easier than average … to predict, thereby suggesting that foreshocks are, in fact, useful in predicting earthquakes.

14. A This paragraph describes a particular application of the theory of earthquake prediction, described in the previous paragraphs, which led to scientists” predicting a large earthquake and saving many lives. Although this is said to have prov[ed] that … earthquake prediction is possible(lines 38–39), it was not a scientific experiment, as there was no control group.

15. C Lines 49–50 mention that seismologists missed predicting the Tangshan earthquake and that over 250,000 people died. This was far worse than the Haicheng earthquake, which was successfully predicted, so that many lives were saved.

16. D The word “evacuation” in line 46 is placed in quotations to indicate that it is not being used in the traditional sense. The task of evacuating a population from a natural disaster does not typically involve showing movies, so doing so is unconventional.

17. C Lines 7–8 say that one of the missionaries who met the ship took us under his wing.

18. E Saying that he could hardly believe that we were really restored to him is like saying he couldn”t believe that we were returned to him.

19. B The narrator states that she could use tools as well as [her] brothers did (lines 20–21), that her first childhood friendship was with a male ship-builder next door, and that she was eager and able to work with the ship-builders around her. Thus, she conveys a clear sense that she considers herself the equal of the males in her life.

20. D The author was emancipated from her confining clothing so that she could work with tools, such as her hatchet, in the shipyard.

21. C The big movements of the day refer to the changes in culture and civilization (line 43).

22. A Choice II is supported by lines 38–40, which say that we had around us the fine flower of New England civilization, as opposed to Michigan, which the author characterizes as the wilderness (line 45). The passage does not suggest that New England had finer gardens or humbler citizens than Michigan had.

23. D The author describes the move to Michigan as a complete upheaval (lines 37–38), and an unwelcome move from the fine flower of New England civilization (lines 39–40), thereby suggesting that she resents the move. She conveys no sign of bewilderment, fear, or awe in this passage, since she describes the move with insight and equanimity.

24. A The passage says that the sisters were so pained by (the lumber wagon”s) appearance that we refused to ride in it (lines 55–56) and that they wanted to look as if we had no association with it (lines 57–58).

Section 7

(Chapter 8, Lesson 1: Solving Equations)

2. C First find out how many cups are in 3 pints.

Set up a ratio to solve for servings:

(Chapter 7, Lesson 4: Ratios and Proportions)

3. A Since the angle shown is a right angle, the arc represents ¼ of the circumference.

(Chapter 10, Lesson 8: Circles)

4. C This question tests your understanding of 30°-60°-90° triangles. The hypotenuse, which corresponds to 2x, is 14. This means that the base is . The height is therefore

(Chapter 10, Lesson 5: Areas and Perimeters)

(Chapter 10, Lesson 3: The Pythagorean Theorem)

5. A Given that ∇x = 3x – 3, find ∇7.

Be careful not to pick answer choice (B) ∇3, because , not 3. Answer choice (A) ∇2 is correct, because .

(Chapter 9, Lesson 1: New Symbol or Term Problems)

6. B A little common sense should tell you that they will not need a full hour to clean the pool, because Stephanie can clean it in an hour all by herself, but Mark is helping. Therefore, you should eliminate choices (C), (D), and (E) right away. You might also notice that it can”t take less than 30 minutes, because that is how long it would take if they both cleaned one pool per hour (so that the two working together could clean it in half the time), but Mark is slower, so they can”t clean it quite that fast. This eliminates choice (A) and leaves (B) as the only possibility.

But you should know how to solve this problem if it were not a multiple-choice question, as well:

Stephanie”s rate for cleaning the pool is one pool per hour. Mark”s rate for cleaning the pool is one pool ÷ 1.5 hours = pools per hour. Combined, they can clean pools per hour. Set up a rate equation using this rate to determine how much time it would take to clean one pool:

(Chapter 9, Lesson 4: Rate Problems)

7. A Change each expression to a base-10 exponential:

(Chapter 8, Lesson 3: Working with Exponentials)

8. B Consider the points (0, 2) and (3, 0) on line l. When these points are reflected over the x-axis, (0, 2) transforms to (0, –2) and (3, 0) stays at (3, 0) because it is on the x-axis. You can then use the slope formula to find the slope of line m:

It”s helpful to notice that whenever a line is reflected over the x-axis (or the y-axis, for that matter—try it), its slope becomes the opposite of the original slope.

(Chapter 10, Lesson 4: Coordinate Geometry)

(Chapter 8, Lesson 5: Factoring)

10. D The total area of the patio to be constructed is . The slab shown in the figure has an area of 8 ft2. Therefore, to fill the patio you will need slabs.

(Chapter 10, Lesson 5: Areas and Perimeters)

11. D The prize money ratio can also be written as 7x:2x:1x. Because the total prize money is $12,000,

(Chapter 7, Lesson 4: Ratios and Proportions)

12. E Always read the problem carefully and notice what it”s asking for. Don”t assume that you must solve for x and y here. Finding the value of is much simpler than solving the entire system:

(Chapter 8, Lesson 2: Systems)

13. D Think carefully about the given information and what it implies, then try to find counterexamples to disprove the given statements. For instance, try to disprove statement I by showing that s can be even. Imagine :

This doesn”t work because r must be an integer. Why didn”t it work? Because 2r must be even, but if s is even, then s + 1 must be odd and cannot equal an even number, so s must always be odd and statement I is true. (Eliminate choice (B).)

Since 1 is an integer, we”ve proven that r is not necessarily even, so II is false. (Eliminate choices (C) and (E).)

Since we still have two choices remaining, we have to check ugly old statement III. Try the values we used before. If and , then , which is an integer. But is it always an integer? Plugging in more examples can”t prove that it will ALWAYS be an integer, because we can never test all possible solutions. We can prove it easily with algebra, though. Since :

Since 2 is an integer, statement III is necessarily true.

(Chapter 9, Lesson 3: Numerical Reasoning Problems)

(Chapter 6, Lesson 7: Thinking Logically)

14. C Find all the possible products of the values on two chips: ; ; ; ; ; ; ; ; ; ; ; ; ; ; . There are 15 different combinations of chips. Of these, only the last 2 yield products that are greater than 20. So the probability is 2/15.

(Chapter 9, Lesson 6: Probability Problems)

15. D In this problem, only the signs of the terms matter. By following the rule of the sequence, you should see that the first six terms of the sequence are +, –, –, +, –, –,…. The pattern {+, –, –} repeats forever. In the first 100 terms, the pattern repeats times. Because each repetition contains two negative numbers, in 33 full repetitions there are negative numbers. The 100th term is the first term of the next pattern, which is positive, so the total number of negative terms is 66.

(Chapter 11, Lesson 1: Sequences)

16. B Draw the five triangles. The simplest way to solve this problem is to compare the choices one pair at a time. For instance, it should be clear just by inspection that and , so we can eliminate A. Similarly, it should be clear that and , so we can eliminate C. Likewise, since RB > RD and SB > SD, we can eliminate D. Finally, we compare B with E. Since RB and RE are each a diagonal of one of the square faces, they must be equal. But SB is clearly longer than SE, because SB is the hypotenuse of triangle SEB, while SE is one of the legs.

(Chapter 10, Lesson 7: Volumes and 3-D Geometry)

(Chapter 6, Lesson 7: Thinking Logically)

Section 8

1. C If the review suggested that the décor of the restaurant was insipid (tasteless), but that the cuisine came close to compensating for it, the review must have been part positive and part negative, that is, ambivalent. indefatigable = untiring; banal = lacking originality; ambivalent = characterized by conflicting feelings; sublime = supreme, impressive; piquant = spicy; tepid = lukewarm

2. C The sentence suggests that Dr. Thompson should have characterized the results as unusual, but didn”t. meticulous = concerned with detail; belligerent = prone to fighting; anomalous = deviating from the norm; convergent = coming together; warranted = appropriate to the situation

3. B They would hope that bad news did not predict further bad news. amalgam = a combination of diverse elements; harbinge = omen; arbiter = judge; talisman = an object with magical power

4. C To bring slaves out of bondage is to free or unfetter them. encumber = burden; forgo = relinquish

5. D A writer who can produce both decorative poetry and a keenly analytical mystery novel is a versatile writer; that is, she is able to write in divergent styles. flamboyant = ornate; immutability = permanence, unchangeability; austere = plain; florid = ornate; grandiloquent = characterized by pompous language

6. B The word because indicates that the sentence shows a cause-and-effect relationship. There are several ways to complete this sentence logically, but the only one among the choices is (B), because multifarious (widely varied) mechanisms would logically “stymie” (impede) scientists who are trying to investigate them. efficacious = capable of producing a desired effect; bilked = cheated; conspicuous = obvious; thwarted = prevented; hampered = hindered; lucid = clear; proscribed = forbidden

7. B If the cultural assumption that there are many alien civilizations … stems in no small way from … the “Drake Equation,” then this equation has had quite an influence on public opinion.

8. E The first two paragraphs discuss how the Drake Equation has led to the belief that there are many alien civilizations in the universe. The third paragraph discusses the author”s contrasting view that there is indeed probably much simple life in the universe but very little if any other complex life.

9. B The sentence states that a planet could go from an abiotic state to a civilization in 100 million years thereby implying that a civilization must, by definition, not be abiotic. Choice (B) is the only choice that necessarily cannot apply to a civilization.

10. A The author states his thesis in lines 38–39: perhaps life is common, but complex life is not, and goes on to explain this thesis, stating in lines 61–67 that research shows that while attaining the stage of animal life is one thing, maintaining that level is quite another…. Complex life is subject to an unending succession of planetary disasters, creating what are known as mass-extinction events.

11. A The phrase the evolutionary grade we call animals refers to the level of life form produced by evolution.

12. C Statement (A) is supported in lines 48–50, statement (B) is supported in lines 74–76, statement (D) is supported in lines 38–39, and statement (E) is supported in lines 51–55.

13. C The sample size of one refers to the uniqueness of Earth history (line 78).

14. A The first quotation in lines 101–103 is described as a rejoinder, or an opposing response, to the author”s thoughts. The author then responds with his own quotation.

15. C The author says that he does not conclude that there are no other cats (Rare Cat Hypothesis), only that there are no other cats exactly like Wookie in order to convey the idea that one should not draw conclusions based on one occurrence.

16. B The author says that life is opportunistic to summarize the next statement that the biosphere has taken advantage of the myriad of strange idiosyncrasies that our planet has to offer.

17. D The passage says that these creatures might naively assume that these qualities, very different from Earth”s, are the only ones that can breed complexity, that is, that all life evolved the same way.

18. A The author of Passage 1 believes that complex life, once evolved, faces numerous dangers that push it toward extinction. The author would point this fact out in response to the statement in lines 134–135 of Passage 2.

19. D The author of Passage 1 says in line 26, In my view, life in the form of microbes or their equivalents is very common in the universe, perhaps more common than even Drake and Sagan envisioned. The author of Passage 2 says in line 139, My bet is that many other worlds, with their own peculiar characteristics and histories, co-evolve their own biospheres. Both authors seem to agree that there is a lot of undiscovered life out there in the universe.

Section 9

1. B When you list items in a sentence, the items should have the same grammatical form. If the first item is in the gerund, they should all be in the gerund. Because the sentence says Eating an english muffin and sitting down, drink coffee should instead be drinking coffee.(Chapter 15, Lesson 3: Parallelism)

2. D The sentence begins with a participial phrase, so the subject of the participle, pretending, must also be the subject of the main clause. Since Mark is the one pretending, the subject of the main clause should be Mark. Choice (C) is incorrect because the pronoun it lacks a proper antecedent and appears to refer, illogically, to the practice. Choice (E) is incorrect because it uses an unidiomatic phrase, convince in letting, rather than the proper idiom, convince to let. (Chapter 15, Lesson 7: Dangling and Misplaced Participles)

3. C The verb are is the improper tense. It should be be as in answer choice (C). (Chapter 15, Lesson 9: Tricky Tenses)

4. C When you list items in a sentence, the items should have the same grammatical form. If the first term is in the noun form, then they all should be in the noun form. Because the sentence says his temper, impatience, how easily he can be irritated should instead be irritability.(Chapter 15, Lesson 3: Parallelism)

5. B Before she gave the gracious speech, she won the match. The verb winning should instead be in the past perfect form , having won.(Chapter 15, Lesson 9: Tricky Tenses)

6. C The sentence begins by describing something that was the most influential science treatise of the 20th century. The pronoun to follow the comma should describe this treatise. Choice (C) corrects the error in the most logical and concise fashion. (Chapter 15, Lesson 7: Dangling and Misplaced Participles)

7. B The pronoun their does not agree in number with its singular antecedent, neither. Choice (B) corrects this error concisely. Choices (C) and (E) are also guilty of pronoun-antecedent disagreement, and choice (D) produces a sentence fragment. (Chapter 15, Lesson 5: Pronoun-Antecedent Disagreement)

8. A The original sentence is best. All other choices are unidiomatic. (Chapter 15, Lesson 10: Idiom Errors)

9. C The sentence requires parallel phrasing of the two things that the storm waves did: inundating and sweeping. Choice (A) is not parallel and is needlessly wordy. Choice (B) is vague, since it does not explain what swept away the cars. In choice (D), the pronoun it does not agree in number with storm waves. Choice (E) misuses the semicolon, because the phrase that follows the semicolon is not an independent clause. (Chapter 15, Lesson 3: Parallelism)

10. B In the original sentence, the verb have been depicted does not agree with its singular subject, life. In choice (C), the phrase depicted with is unidiomatic, and the verb has had depicted is illogical. In choice (D), the pronoun they lacks a clear and logical antecedent. The logic and phrasing in choice (E) is awkward. (Chapter 15, Lesson 1: Subject-Verb Disagreement

11. D In the original sentence, the modifying phrase at the age of seven is misplaced, and incorrectly implies that the speaker”s father, rather than the speaker himself, was seven. Choices (B), (C), and (E) commit the same error, but in slightly different ways. (Chapter 15, Lesson 7: Dangling and Misplaced Participles)

12. A The original sentence is best, since it uses concise and logical parallel phrasing. (Chapter 15, Lesson 3: Parallelism)

13. E The pronoun them refers to a plural subject. However, anyone is singular. Answer choice (E) clears up this pronoun-antecedent disagreement in the most concise and logical way. (Chapter 15, Lesson 5: Pronoun-Antecedent Disagreement)

14. A Although the original phrasing is not the most concise option, it is the only one that logically coordinates the ideas in the sentence.

1Those who believe in absolute good and evil

1One who fights for a cause; also, a prize fighter